10
$\begingroup$

Let $X\ge 2$ be large. Let $$A = \left\{\frac{a}{q}: 1\le a < q\le X,\ (a, q) = 1\right\},$$ and let $$B = \left\{\frac{a}{p}: 1\le a < p\le X,\ (a, p) = 1,\text{ and $p$ is prime}\right\}.$$ Note that for distinct $\frac{a}{q}, \frac{a'}{q'}\in A$, we have $$\left\lVert \frac{a}{q} - \frac{a'}{q'}\right\rVert\ge\frac{1}{qq'}\ge\frac{1}{X^2},\quad\text{where}\quad\lVert\beta\rVert := \min_{n\in\mathbb{Z}} |\beta - n|.$$ It follows that in any interval of length $X^{-2}$ in $[0, 1]$, there are at most $O(1)$ elements of $A$, and this is clearly the best result possible, since $|A|\gg X^2$. Is it possible to get better results for $B$, since we have that $|B|\asymp X^2 / \log X$ by the prime number theorem. In particular, is it possible to get a bound of $o(\log X)$ for the number of elements of $B$ in an interval of length $$|B|^{-1}\ll X^{-2}\log X?$$ Note that the inclusion $B\subseteq A$ gives the trivial bound $O(\log X)$.

$\endgroup$
8
  • $\begingroup$ With the current definition, we have $B = \{ A\}$, are you sure that is what you want? $\endgroup$
    – Dirk
    Jul 10, 2018 at 10:51
  • $\begingroup$ @Dirk, I'm guessing OP has forgotten to tell us that $p$ is restricted to being a prime. $\endgroup$ Jul 10, 2018 at 12:35
  • 1
    $\begingroup$ I've corrected these typos. The inequality was originally the wrong way around. $\endgroup$ Jul 10, 2018 at 19:50
  • 1
    $\begingroup$ I wonder whether the paper Harman, Glyn, Numbers badly approximable by fractions with prime denominator, Math. Proc. Cambridge Philos. Soc. 118 (1995), no. 1, 1–5, MR1329453 (96c:11080) is relevant here. $\endgroup$ Jul 11, 2018 at 0:45
  • 2
    $\begingroup$ This type of inequality on fractions is the starting point of the large sieve inequality. In case that you are actually interested in applications with the large sieve: Wolke observed that the sieve estimates with prime denominators can almost save the log factor: Wolke, D. On the large sieve with primes. Acta Math. Acad. Sci. Hungar. 22 (1971/72), 239–247. mathscinet.ams.org/mathscinet-getitem?mr=291121 $\endgroup$ Jul 17, 2018 at 21:43

1 Answer 1

1
$\begingroup$

Wolke (On the large sieve with primes, Acta Math. Acad. Sci. Hungar. 22 (1971/72), 239–247, MathSciNet MR0291121 (45 #215)) has worked on this question. His motivation was Gallagher's approach to the large sieve. If I remember correctly, his result was almost saving a factor $\log x$, which is what one would expect.

$\endgroup$

Your Answer

By clicking “Post Your Answer”, you agree to our terms of service and acknowledge you have read our privacy policy.

Not the answer you're looking for? Browse other questions tagged or ask your own question.